Đến nội dung

moonkey01

moonkey01

Đăng ký: 22-05-2016
Offline Đăng nhập: 17-09-2018 - 20:07
****-

#667102 VMF's Marathon Hình học Olympic

Gửi bởi moonkey01 trong 05-01-2017 - 17:10

Em giải thích rõ hơn phần tính toán để có được $IN||AF$.

 

Đặt $BC=a,CA=b,AB=c(b\neq c)$ thì dễ dàng có $\frac{IE}{IA}=\frac{a}{b+c}$. Mặt khác cũng tính được $BE=\frac{ca}{c+b}$ nên $NE=BN-BE=\frac{a}{2}-\frac{ca}{c+b}=\frac{a(b-c)}{2(b+c)}$. Hơn nữa $NF=NC-CF=\frac{a}{2}-\frac{c+a-b}{2}=\frac{b-c}{2}$ nên $\frac{NE}{NF}=\frac{a(b-c)}{2(b+c)}:\frac{b-c}{2}=\frac{a}{b+c}=\frac{IE}{IA}$. Theo định lý Thales đảo, $IN||AF$.

 

Bài toán 103 (Mathematical Refections 6/2014). Cho tam giác $ABC$ nội tiếp $(\Gamma)$ có $M$ là điểm chính giữa cung $BC$ không chứa $A$. Gọi $l_{b},l_{c}$ là đường thẳng qua $B,C$ song song $AM$ và cắt lại $(\Gamma)$ tại $P,Q$. $PQ$ cắt $AB,AC$ tại $X,Y$. $AM$ cắt lại $(AXY)$ tại $N$. Chứng minh rằng trung trực $BC,XY,MN$ đồng quy.




#667097 VMF's Marathon Hình học Olympic

Gửi bởi moonkey01 trong 05-01-2017 - 15:51

 

Bài toán 102 (Tập huấn đội IMO 2016)Cho tam giác $ABC$ nội tiếp đường tròn $(O)$ cố định. $B$, $C$ cố định, $A$ di chuyển trên $(O)$. $I$ là tâm đường tròn nội tiếp tam giác $ABC$. $K$, $L$ theo thứ tự là trực tâm các tam giác $IAB$, $IAC$. $P$ đối xứng với $O$ qua trung điểm $KL$. Chứng minh rằng $AP$ đi qua một điểm cố định khi $A$ thay đổi.

 

Lời giải của em như sau.

 

Trước hết, ta phát biểu hai bổ đề sau:

 

Bổ đề 1: Hai tam giác đồng dạng có hai cặp cạnh tương ứng vuông góc thì cặp cạnh còn lại cũng vuông góc.

 

Bổ đề 2: $KL,AF$ vuông góc trên $(I)$ với $F$ là tiếp điểm đường tròn bàng tiếp góc $\angle BAC$ của tam giác $ABC$.

 

Hai bổ đề trên đều quen thuộc nên không trình bày lại phép chứng minh tại đây, riêng bổ đề 2 chính là hệ quả bài toán 3 trong kỳ thi Olympic Sharygin vòng 1 năm 2017. Quay trở lại bài toán ban đầu. Do trường hợp $AB=AC$ là hiển nhiên nên không mất tính tổng quát, ta xét trường hợp $AB<AC$ và vị trí các điểm như hình vẽ.

 

Do $K,L$ là trực tâm các tam giác $IAB,IAC$ và $I$ là tâm nội tiếp tam giác $ABC$ nên ta có:

$\angle KAL=\angle KAB+\angle LAC-\angle BAC=90^{o}-\frac{\angle ABC}{2}+90^{o}-\frac{\angle BCA}{2}-\angle BAC=90^{0}-\frac{\angle BAC}{2}$

 

Gọi $Q$ là điểm sao cho $AQ\perp BC$ và $QP||AL$ thì ta có:

$\angle KAQ=\angle BAK-\angle BAQ=90^{o}-\frac{\angle ABC}{2}-(90^{o}-\angle ABC)=\frac{\angle ABC}{2}=\angle IBC$.

 

Mặt khác do $\angle AKQ=180^{o}-\angle KAL=180^{o}-(90^{o}-\frac{\angle BAC}{2})=90^{o}+\frac{\angle BAC}{2}=\angle BIC$ nên tam giác $AKQ$ đồng dạng tam giác $BIC$ (g.g). Gọi $M,N$ là trung điểm $AQ,BC$ thì tam giác $AKM$ đồng dạng tam giác $BIN$ (c.g.c). Từ bổ đề dễ dàng có $KM\perp IN$, lại có $KL\perp AF$ nên ta chỉ cần chứng minh rằng $IN||AF$. Nhưng điều này đúng do dễ dàng tính toán theo 3 cạnh tam giác $ABC$ bởi tính chất phân giác để có tỉ lệ $\frac{IE}{IA}=\frac{BC}{CA+AB}=\frac{NE}{NF}$. Từ đó dẫn đến $KM\equiv KL\perp IN$, lại có $\Delta MAL=\Delta MQK$ (g.c.g) nên $ML=MK$ hay $M$ là trung điểm $KL$ hay $M$ cũng là trung điểm $OP$. Lại có $\frac{AQ}{BC}=\frac{AK}{BI}=cot\frac{\angle BAC}{2}$ không đổi nên $AQ$ không đổi. Gọi $R$ là đối xứng của $P$ qua $A$ thì $OR=AQ$ không đổi, lại có $OR||AP\perp BC$ nên $AP$ luôn qua $R$ cố định $\blacksquare$

 

Thầy có lời giải nào mà để tránh tính toán phần sau không ạ ?

 

Hình vẽ bài toán:

Hình gửi kèm

  • Ảnh chụp Màn hình 2017-01-05 lúc 15.52.45.png



#667068 Đề Thi VMO năm 2017

Gửi bởi moonkey01 trong 05-01-2017 - 13:13

Ý bạn là sao ? 

 

Do $B(x)$ có 2 nghiệm nên em không rõ là $Q(x)-(3x-1)$ có nhận số còn lại làm nghiệm hay không (em không rành lắm về đa thức). Nếu có, anh có thể chứng minh chặt chẽ luôn được không ạ ? 




#666975 VMF's Marathon Hình học Olympic

Gửi bởi moonkey01 trong 04-01-2017 - 21:05

Lời giải sau do bạn Nguyễn Lê Phước gửi tới tác giả.

 

attachicon.gifFigure4051.png

 

Lời giải bài toán 101. Gọi $NI$ cắt $HK$ tại $S$, $MI$ cắt $HL$ tại $T$ thì $S,T$ là tâm bàng tiếp tam giác $ANH,AMH$. Từ đó $\angle ASE=45^\circ=\angle AHL$ và $\angle AES=90^\circ+\angle ANE=\angle ALH$. Từ đó hai tam giác $ASE$ và $ALH$ đồng dạng g.g. Lại có $\angle ATH=90^\circ-\frac{\angle AMN}{2}=180^\circ-\angle AIN=\angle AIS$. Từ đó hai tam giác $ATH$ và $AIS$ đồng dạng g.g. Vậy $\frac{ES}{EI}=\frac{LH}{LT}$. Tương tự $\frac{FT}{FI}=\frac{KH}{KS}$. Từ đó $\frac{FI}{FT}.\frac{LT}{LH}=\frac{EI}{ES}.\frac{KS}{KH}$. Dùng định lý Menelaus dễ thấy $FL,ES$ cùng đi qua một điểm trên $IH$.

 

Lời giải khác của bạn Nguyễn Đức Bảo có thể xem tại http://artofproblems...1308940p7009306, bài toán này nằm trong chuỗi bài mở rộng bài toán IMO 2009 của tác giả http://analgeomatica...-2009-ngay.html

 

Bài toán 102 (Tập huấn đội IMO 2016)Cho tam giác $ABC$ nội tiếp đường tròn $(O)$ và $P$ di chuyển trên cung $BC$ không chứa $A$. Đối xứng của $PA$ qua $PB$, $PC$ lần lượt cắt $AB$, $AC$ tại $F$, $E$. Một đường thẳng vuông góc với $PA$ tại một điểm chia $PA$ theo tỉ số cố định cắt tiếp tuyến tại $A$ của $(AEF)$ tại $Q$. Chứng minh rằng $Q$ luôn thuộc đường thẳng cố định khi $P$ thay đổi.

 

Trong quá trình cố gắng chứng minh bài toán, em phát hiện ra một số tính chất khác như sau. Không mất tính tổng quát, ta giả sử rằng $AB<AC$.

 

Tính chất 1: $EF$ đi qua $D$ là chân đường phân giác ngoài từ $A$ của $\Delta ABC$.

 

Thật vậy, do $NE$ là phân giác $\angle ANC$ nên $\frac{EC}{EA}=\frac{NC}{NA}=\frac{AC}{AH}$. Tương tự ta có $\frac{FA}{FB}=\frac{AH}{AB}$ nên nếu gọi $AD$ là phân giác ngoài của $\Delta ABC$ thì theo định lý Menelaus, chứng minh được $D,E,F$ thẳng hàng.

 

Tính chất 2: $NF,ME,AI$ đồng quy.

 

Từ tính chất 1 và lưu ý rẳng $AI$ là phân giác $\angle BAC$, lại có $AB,AC$ là hai đường đẳng giác trong $\Delta AMN$ nên gọi $AI$ cắt $BC$ tại $P$ thì $(DP,BC)=(DP,NM)=-1$. Mặt khác gọi $AI$ cắt $EF$ tại $S$ thì $A(DP,BC)=A(DS,FE)=(DS,FE)=-1=(DP,NM)$ nên $SP,NF,ME$ đồng quy hay $NF,ME$ cắt nhau trên $AI$.




#666626 VMF's Marathon Hình học Olympic

Gửi bởi moonkey01 trong 02-01-2017 - 14:30

Em đề nghị bài toán tiếp theo, đã được phát biểu lại cho gọn hơn. Bài toán trích từ đề thi chọn HSG lớp 9 trường THPT chuyên Trần Đại Nghĩa năm ngoái.

 

Bài toán 99: Cho tam giác $ABC$ có $O$ là tâm ngoại tiếp. Đường thẳng $d$ đi qua $O$ sao cho $B,C$ nằm cùng phía với $d$. Gọi $M,N$ là hình chiếu của $B,C$ trên $d$. Đường thẳng qua $M$ vuông góc $CA$ cắt đường thẳng qua $N$ vuông góc $AB$ tại $T$. Chứng minh rằng $T$ luôn thuộc đường tròn cố định khi $d$ thay đổi.

 

Theo cái nhìn chủ quan của em thì đây là một bài toán có cấu hình đẹp và thú vị, nên em rất mong thầy Hùng và các anh chị đóng góp những mở rộng của bài toán trên.




#666595 VMF's Marathon Hình học Olympic

Gửi bởi moonkey01 trong 02-01-2017 - 12:18

Bài toán 98. Cho tam giác $ABC$ giả sử có điểm $P$ nằm trong tam giác sao cho $\angle PBA=\angle PCA$ đồng thời nếu có các đường tròn $(K)$ qua $P,C$ và đường tròn $(L)$ qua $P,B$ sao cho $(K),(L)$ cắt nhau tại $Q$ khác $P$ thì $BQ$ đi qua giao điểm $E$ của $(K)$ và $AC$ còn $CQ$ đi qua giao điểm $F$ của $(L)$ và $AB$. Chứng minh rằng $BE=CF$.

 

attachicon.gifFigure4229.png

 

Nguồn gốc: http://www.cut-the-k.../Stoyanov.shtml

 

Lời giải của em như sau:

 

Không mất tính tổng quát, giả sử vị trí các điểm như hình vẽ thì ta có $\angle BFP = \angle BQP = \angle PCA = \angle PBA$ nên $\Delta PBF$ cân tại $P$ hay $PB=PF$. Tương tự ta có $PE=PC$ nên $\Delta PBE=\Delta PFC$ (c.g.c), dẫn đến $BE=CF$.




#662951 $\sum \frac{ab}{3+bc}\leq\frac...

Gửi bởi moonkey01 trong 24-11-2016 - 21:29

Cho $a,b,c\geq 0$ thoả mãn $a+b+c=3$. Chứng minh rằng $\sum \frac{ab}{3+bc}\leq\frac{3}{4}$




#662222 Tạp chí PI của bạn - Thách đấu Toán học số 1

Gửi bởi moonkey01 trong 17-11-2016 - 14:31

Các bài toán đều hay và đẹp ạ  :D  :D

 

Em hy vọng tạp chí PI sẽ thành công như mong đợi  :) !




#658562 Đề chọn đội tuyển học sinh giỏi quốc gia tỉnh Vĩnh Phúc (ngày 2) 2016-2017

Gửi bởi moonkey01 trong 20-10-2016 - 19:38

Nói như Hoàng thì bảng đã cho là $89$x$89$ ?

P/s: Mình cũng làm sai rồi. Cột chính giữa là cột 50... Và cách làm của Hoàng cũng chỉ đúng cho bảng có cạnh là một số chia 8 dư 7 thôi... Ở đây mình cần cách chứng minh bảng có cạnh chia 8 dư 3 không phủ được...

 

Cảm ơn bạn đã góp ý, đúng là mình có sơ suất. Một lời giải khác thay thế cho lời giải sai ở trên:

 

Từ góc trên bên trái bảng, ta tô màu các nhóm ô vuông $2.2$ xen kẽ trắng-đen, với góc trên bên trái màu trắng. Số ô đen nhiều hơn số ô trắng là $4$ ô, mỗi con tetramino phủ $2$ ô trắng và $2$ ô đen, vậy câu trả lời là phủ định. 




#658467 ĐỀ THI HỌC SINH GIỎI TỈNH BÌNH THUẬN VÒNG 1 2016-2017

Gửi bởi moonkey01 trong 19-10-2016 - 20:19

Bài 2:

 

Áp dụng bất đẳng thức Cauchy-Schwarz: $\sum \frac{x^2}{y+z}\geq \frac{(x+y+z)^2}{2(x+y+z)}=\frac{x+y+z}{2}$

 

Áp dụng bất đẳng thức AM-GM: $\sum \frac{xy}{x+y}\leq\sum \frac{(x+y)^2}{4}.\frac{1}{x+y}=\frac{x+y+z}{2}$

 

Kết hợp 2 bất đẳng thức trên suy ra điều phải chứng minh.

 

Bài 5:

 

Theo định lý Ceva dạng lượng giác thì $\frac{sin\angle MAB}{sin\angle MAC}.\frac{sin\angle MBC}{sin\angle MBA}.\frac{sin\angle MCA}{sin\angle MCB}=-1\Leftrightarrow \frac{sin\angle A_{1}AC}{sin\angle A_{1}AB}.\frac{sin\angle B_{1}BA}{sin\angle B_{1}BC}.\frac{sin\angle C_{1}CB}{sin\angle C_{1}CA}=-1$

 

Do đó $AA_{1},BB_{1},CC_{1}$ đồng quy.




#658397 ĐỀ THI HSG THPT CHUYÊN VÀ CHỌN ĐỘI TUYỂN DỰ THI HSG QUỐC GIA THPT NĂM 2016-20...

Gửi bởi moonkey01 trong 19-10-2016 - 08:51

Bài 6, với ý tưởng từ anh UphluMuach:

 

Ta sẽ chỉ ra rằng $n=15$ là giá trị nhỏ nhất của $n$ để thỏa mãn bài toán.

 

Thật vậy, với $n=12$ thì ta phân hoạch $A$ thành $4$ tập con là $\left \{ 1,7,10 \right \};\left \{ 2,8,11 \right \};\left \{ 3,6,12 \right\};\left \{ 4,5,9 \right \}$.

 

Với $n=13$ thì ta phân hoạch $A$ thành $4$ tập con là $\left \{ 1,6,7,13 \right \};\left \{ 2,9,11 \right \};\left \{ 3,5,10 \right \};\left \{ 4,8,12 \right \}$.

 

Với $n=14$ thì ta phân hoạch $A$ thành $4$ tập con là $\left \{ 2,10,12 \right \};\left \{ 3,8,11 \right \};\left \{ 1,6,7,13 \right \};\left \{ 4,5,9,14 \right \}$.

 

Với $n=15$, giả sử không thỏa mãn, ta xét tập hợp $C=\left \{ 7,8,9,...,15 \right \}$ có $9$ phần tử, mà $3$ phần tử bất kỳ nào cũng là độ dài $3$ cạnh của một tam giác, ngoại trừ bộ $(7,8,15)$. Theo nguyên lý Dirichlet, khi phân hoạch $A$ thành $4$ tập con thì phải có $1$ tập chứa $3$ phần tử của $C$, nên để không có $3$ số nào hợp thành $3$ cạnh của tam giác thì bộ số đó phải là bộ $(7,8,15)$ và mỗi tập còn lại trong họ $B$ chứa đúng 2 phần tử trong $C$. Lưu ý thêm rằng không có tập nào trong họ $B$ có hơn $4$ phần tử, nếu không sẽ có mâu thuẫn). Còn lại $3$ tập trong họ $B$, xét hiệu của $2$ phần tử lớn nhất, mỗi hiệu đó không vượt quá $14-9=5$ và phải không nhỏ hơn phẩn tử lớn thứ ba, mà phần tử lớn nhất trong các phần tử lớn thứ ba trong mỗi tập đó phải không nhỏ hơn $6$ (do lấy từ $1$ đến $6$) nên điều giả sử là sai. 

 

Kết luận: Giá trị nhỏ nhất của $n$ là $15$.




#658297 ĐỀ THI HSG THPT CHUYÊN VÀ CHỌN ĐỘI TUYỂN DỰ THI HSG QUỐC GIA THPT NĂM 2016-20...

Gửi bởi moonkey01 trong 18-10-2016 - 17:04

Bài 4: Rõ ràng $p>3$ thì bài toán mới có nghĩa.

 

Từ giả thiết, ta suy ra $(a-b)(a^2+ab+b^2)\vdots p$, mà $a\neq b$ và $a,b<p$ nên $a^2+ab+b^2\vdots p$, tương tự có $b^2+bc+c^2$ và $c^2+ca+a^2$ là bội của $p$. Từ đó $a^2+ab+b^2-b^2-bc-c^2\vdots p$ hay $(a-c)(a+b+c)\vdots p$, lại có $a\neq c$ và $a,c<p$ nên $a+b+c\vdots p$.

 

Do $a+b+c<3p$ nên ta xét các trường hợp:

 

- $a+b+c=p$:

 

Ta có $a+b\equiv -c$ (mod $p$) nên $a^2+2ab+b^2\equiv c^2$ (mod $p$) hay $(a^2+ab+b^2)+ab+bc+ca\equiv c(a+b+c)$ (mod $p$).

Từ đó $ab+bc+ca\vdots p$. Để ý rằng $(a+b+c)^2=a^2+b^2+c^2+2(ab+bc+ca)$ nên $a^2+b^2+c^2\vdots p$ hay $a^2+b^2+c^2\vdots a+b+c$.

 

- $a+b+c=2p$:

 

Tương tự trên, ta có $a^2+b^2+c^2\vdots p$, mà $a^2+b^2+c^2\vdots 2$ và $(p,2)=1$ nên $a^2+b^2+c^2\vdots a+b+c$.




#658296 ĐỀ THI HSG THPT CHUYÊN VÀ CHỌN ĐỘI TUYỂN DỰ THI HSG QUỐC GIA THPT NĂM 2016-20...

Gửi bởi moonkey01 trong 18-10-2016 - 16:42

Bài 3:

 

Xét trường hợp $AB<AC$, các trương hợp khác chứng minh tương tự.

 

a/ $\bigtriangleup EXY$ có $XB$, $YC$ là hai đường cao nên nếu gọi $M'$ là trung điểm $XY$ thì theo một bài toán quen thuộc, $M'B$ và $M'C$ là hai tiếp tuyến của $(EBC)$, dẫn đến $M\equiv M'$ hay $MX=MY$.

 

b/ Có biến đổi tỉ số $\frac{AB}{AC}=\frac{DB}{DC}=\frac{BY}{CX}$, lại có $\angle ABE=\angle ACE$ dẫn đến $\angle ABY=\angle ACX$ hay $\bigtriangleup ABY\sim \bigtriangleup ACX$. Từ đó $\angle XAY=\angle XAB+\angle BAY=\angle XAB+\angle CAX=\angle XEY$ nên $AEXY$ nội tiếp. Lại có $\angle DAE=90^{o}$ nên $AE,BC,XY$ đồng quy tại tâm đẳng phương của $(ABC), (BCXY), (AEXY)$ hay $N$ di chuyển trên $BC$ cố định.




#658293 ĐỀ THI HSG THPT CHUYÊN VÀ CHỌN ĐỘI TUYỂN DỰ THI HSG QUỐC GIA THPT NĂM 2016-20...

Gửi bởi moonkey01 trong 18-10-2016 - 16:15

Bài 2:

 

Bằng phép quy nạp đơn giản, ta có $x_{n}>2 \forall n \in \mathbb{N}, n\geq 1$.

 

Xét các khả năng sau:

 

- Nếu $a=6$ thì cũng bằng phép quy nạp, ta suy ra $x_{n}=6 \forall n \in \mathbb{N}, n\geq 1$ hay dãy đã cho hội tụ về $6$.

 

- Nếu $2<a<6$ thì ta chứng minh đây là dãy số tăng, thật vậy ta có $x_{n+1}>x_{n}\Leftrightarrow \frac{1}{2}x_{n}+\sqrt{2x_{n}-3}>x_{n}\Leftrightarrow (x_{n}-2)(x_{n}-6)<0$, đúng. Dãy này tăng và bị chặn trên bởi $6$ nên có giới hạn hữu hạn. Gọi giới hạn này là $l$, chuyển đẳng thức truy hồi về theo $l$ và giải phương trình, ta nhận $l=6$.

 

- Nếu $a>6$ thì ta chứng minh đây là dãy số giảm tương tự như trường hợp trên, dãy bị chặn dưới bởi $6$ nên cũng có giới hạn hữu hạn $l$, và cũng tính được $l=6$.

 

Kết luận: $lim(x_{n})=6$




#658291 ĐỀ THI HSG THPT CHUYÊN VÀ CHỌN ĐỘI TUYỂN DỰ THI HSG QUỐC GIA THPT NĂM 2016-20...

Gửi bởi moonkey01 trong 18-10-2016 - 16:03

Bài 1:

 

Hệ phương trình đã cho tương đương với $\left\{\begin{matrix} 16x^3+24x^2+12x+3=\sqrt[3]{y}\\ (x-y)[(4x+2y+3)^2+3(2y+1)^2]=\sqrt[3]{y}-\sqrt[3]{x} \end{matrix}\right.$

 

Xét phương trình thứ hai, nếu $x>y$ thì $\sqrt[3]{y}\geq\sqrt[3]{x}$ hay $y\geq x$. Tương tự thì ta phải có $x=y$.

 

Quay lại phương trình đầu tiên, khi $x=y$ thì $2(2x+1)^{3}+(2x+1)=2x+\sqrt[3]{x}$. Khi đó $2x+1=\sqrt[3]{x}$

 

Phương trình này cho nghiệm duy nhất $x=-1$. Vậy nghiệm của hệ là $x=y=-1$